Você está na página 1de 27

CAS Exam ST

Spring 2014 Exam

last update - May 2, 2014

Exam Notes:

You have 2 hour and 30 minutes to complete the exam of 25 questions.

Each correct answer is worth 2 points.

Each wrong answer is worth -0.5 points.

Each question left blank is worth 0 points.

Make sure you have the ST exam tables on hand before you start the exam.

CAS ST Spring 2014 Exam


1. You are given the following information for a policy covering two types of claims:

Total number of claims is given by a Poisson process with claims intensity (t) = 10t, t > 0.

At any time the probability of a claim being from Claim Type A is 0.6 and from Claim Type B is
0.4.

Frequency and severity of claims are independent.

Claim severities follow the distributions given in the tables below


Claim Type A Claim Type B
Claim Amount Probability Claim Amount Probability
< 500 0.3 < 1000 0.1
At least 500, but less than 1000 0.5 At least 1000, but less than 2000 0.6
At least 1000 0.2 At least 2000 0.3

Calculate the probability that by time 0.5 there will be fewer than two claims with severity at least
equal to 1000.

A. Less than 0.55


B. At least 0.55, but less than 0.65
C. At least 0.65, but less than 0.75
D. At least 0.75, but less than 0.85
E. At least 0.85

What is the proability that an occuring claim, X, has severity at least equal to 1000? Compute this by
conditioning on the type of the claim.

P[X 1000] = P[X 1000|Type A]P[Type A] + P[X 1000|Type B]P[Type B]


= 0.2 0.6 + (0.6 + 0.3) 0.4 = 0.48.

The number of claims by time 0.5 is therefore a Poisson random variable with parameter
Z 0.5 0.5 !
10t2
(0.48) 10tdt = (0.48) = 0.6.
0 2 0

The probability that this is fewer than 2 is the probability that it is equal to zero or one, e0.6 +0.6e0.6 =
0.878098618 , which fits into the range of answer choice E.

CAS ST Spring 2014 Exam


2. You are given the following information:

Buses arrive according to a Poisson process at a rate of 5 per hour.

Taxis arrive according to a Poisson process at a rate of 10 per hour.

The arrival of buses and taxis are independent.

You get a ride to work from either a bus or a taxi, whichever arrives first.

Calculate the probability you will have to wait more than 10 minutes for a ride to work.

A. Less than 0.05


B. At least 0.05, but less than 0.10
C. At least 0.10, but less than 0.15
D. At least 0.15, but less than 0.20
E. At least 0.20

The combination of two independent Poisson processes is a new Poisson process with rate equal to the
sum of the rates of the component processes. That means that some kind of vehicle arrives as Poisson
Process with rate 15. The probability that the first waiting time from this process is more than 10
minutes (= 1/6 of an hour) is the probability that an exponential random variable with mean 1/15 is
more than 1/6. This is
P[X > 1/6] = e(1/6)/(1/15) = 0.082084999 .
This fits into the range of answer choice B. Note that this answer also seems reasonable, since we expect
a Bus about every 12 minutes and a taxi about every 6 minutes, making it relatively unlikely that the
wait would be longer than 10 minutes for some kind of vehicle to arrive.

CAS ST Spring 2014 Exam


3. Let S(t) = X1 + X2 + . . . + XN (t) be a compound Poisson process where:

Xi is uniform on [0, 3.14]

N (t) is a Poisson process with rate (t) = et , t > 0

Calculate E[S(4)2 ]

A. Less than 2.5


B. At least 2.5, but less than 3.5
C. At least 3.5, but less than 4.5
D. At least 4.5, but less than 5.5
E. At least 5.5

The second moment of the process up to time 4 is the variance plus the first moment squared. We recall
that
Z 4   
t 4 3.14
E[S] = e dt E[X] = (1 e ) = 1.541244447,
0 2
Z 4
(0 3.14)2
  
t 2 4
Var[S] = e dt E[X ] = (1 e ) = 3.226338376.
0 3

If we do not remember the second moment of the uniform, we can compute


Z 3.14  !
3 3.14 3.143

1 1 t 1
E[X 2 ] = t2 dt = = = 3.28653.
0 3.14 3.14 3 0 3.14 3

The second moment of the compound process, then, is 3.226338376 + 1.5412444472 = 5.601772821 ,
answer choice E.

CAS ST Spring 2014 Exam


4. You have a random sample of five observations:

2.5 7.5 12.5 16.0 17.5

The probability density function below is fit to the random sample.


 2
2
xi
f (x) = 2 xi e

Calculate the maximum likelihood estimate of

A. Less than 10
B. At least 10, but less than 11
C. At least 11, but less than 12
D. At least 12, but less than 13
E. At least 13

We recognize this density as Weibull with = 2. Knowing that this is Weibull isnt that helpful unless
you have memorized the MLE for the Weibull. Compute it as usual, via the likelihood and log likelihood
functions. The sum of the squared observations is 781.
 5 Y
2 2 Xi2
Xi e(1/ )
P
L() =
2
P 2
Y Xi
`() = 5 ln(2) 2 5 ln() + ln( (Xi ))
2
P 2
25 X
`0 () = 0 +0+2 3 i

781 2 5
0=2
3
0 = 2 781 2 5 2
r
781
= = 12.49799984 .
5

This is answer choice D.

CAS ST Spring 2014 Exam


5. You are given the following information:

A random variable, X, follows a two-parameter Pareto distribution with parameters = 5 and


unknown.

A random sample of 15 independent observations is taken.

is estimated as cX, where c is a constant.

Calculate the value of c that minimizes the mean squared error of the estimator of .

A. Less than 1.0


B. At least 1.0, but less than 2.0
C. At least 2.0, but less than 3.0
D. At least 3.0, but less than 4.0
E. At least 4.0

It is a bit shorter to compute directly, but in any case you should skip this one on the first pass.

MSEcX () = E (cX )2
 

= E c2 X 2 2cX + 2
 

= c2 E[X 2 ] 2cE[X] + 2

2E[X]
This is a quadratic equation in c, so its minimium is at the vertex, where c = 2E[X 2 ]
. We need the first
and second moments of the Pareto, computing from the tables.

2 2 2
E[X] = = , E[X 2 ] = = .
51 4 (5 1)(5 2) 6
2
 2 2
10
Var[X] = = .
6 4 96

Now we can compute


1
E[X] = nE[X] =
n 4
 2
2 Var(X)
2 102 2 52
E[X ] = Var(X) + E[X] = + = + = .
n 4 96 15 16 72
Finally,
2E[X] /4 72
c= 2
= 52 = = 3.6 ,
2E[X ] 20
72
which corresponds to answer choice D.

If you only recall that the MSE is the variance plus the square of the bias, then the variance and bias
are
 c2 c2 Var(X)
Var cX = 2 n Var(X) = ,
n n

CAS ST Spring 2014 Exam


  c
biascX () = E cX = nE[X] = cE[X] .
n

Finally, the mean squared error (recalling that n = 15) is

c2 Var(X)
MSE(cX) = + (cE[X] )2
n
2
c2 102


= + c
96 15 4
 2 2

c c c
= 2 + 2 +1
144 16 4
 2 
5c c
= 2 +1
72 2

To minimize this,

5c2
 
d c 10 1
+1 =c
dc 72 2 72 2
10 1
0= c
72 2
1
2 72
c= 10 = = 3.6.
72
20

CAS ST Spring 2014 Exam


6. You are given the following probability distribution:

X P(X = x)
0
1 2
2 1 3

A random sample was taken with the following results:

The value 0 was observed 2 times.

The value 1 was observed 5 times.

The value 2 was observed 3 times.

Calculate the maximum likelihood estimate of .

A. Less than 0.15


B. At least 0.15, but less than 0.18
C. At least 0.18, but less than 0.21
D. At least 0.21, but less than 0.24
E. At least 0.24

Full steam ahead, and thank you very much.

L() = (2 )(2)5 (1 3)3 = 25 7 (1 3)3


`() = 5 ln(2) + 7 ln() + 3 ln(1 3)
7 3
`0 () = 0 + + (0 3)
1 3
7 9
0=
1 3
0 = 7(1 3) 9
0 = 7 21 9
= 7/30 = 0.233

This is answer choice D.

CAS ST Spring 2014 Exam


7. You are given a distribution with the following probability density function, where is unknown:

f (x) = x1 , 0 < x < 1, > 0

and a random sample of five observations

0.13 0.42 0.60 0.71 0.88

Calculate the maximum likelihood estimate of .

A. Less than 2
B. At least 2, but less than 4
C. At least 4, but less than 6
D. At least 6, but less than 8
E. At least 8

Sir, yes, sir!


Y
L() = 5 Xi1
X
`() = 5 ln() + ( 1) ln(Xi )
5 X
`0 () = + ln(Xi )

5 X
0= + ln(Xi )

5 5
= P = = 1.285720299
ln(Xi ) 3.8888707

This fits into answer choice A.

CAS ST Spring 2014 Exam


8. You are given the following:

X is a uniformly distributed random variable with the pdf


1
fX (x) = 0x

= cXmax where Xmax is the largest Xi from a sample of size n.

Determine the value of c that makes an unbiased estimator of when n = 100.

A. Less than 1.005


B. At least 1.005, but less than 1.025
C. At least 1.025, but less than 1.045
D. At least 1.045, but less than 1.065
E. At least 1.065

To find the expected value of the maximum, start with the CDF.
 x n
P[Xmax x] = (P[X x])n =

 x n1  1  nxn1
fXmax (x) = n =
n
Z
nxn1 cn n cn n+1
Z
cn
E[cXmax ] = c x n
= n
x dx = n
=
0 0 n+1 n+1

To make the estimator unbiased, then, take c = (n + 1)/n = 101/100 = 1.01 , which fits into the range
of answer choice B.

CAS ST Spring 2014 Exam


9. You are given the following information about the number of auto claims:

Age Group
Gender Young Old Total
Male 20 15 35
Female 12 22 34
Total 32 37 69

A Chi-squared hypothesis test is performed to determine if gender and age group are independent.

Calculate the p-value of this test.

A. Less than 0.005


B. At least 0.005, but less than 0.010
C. At least 0.010, but less than 0.025
D. At least 0.025, but less than 0.050
E. At least 0.050

The overall proportion of males is 35/69 = 0.507246377. Store this in the x variable of your calculator.
Now create the table of expected outcomes via the uniform distribution, if gender and age group are
independent.

Expected Age Group


Gender Young Old
Male 32x = 16.232 37x = 18.768
Female 32(1-x) = 15.768 37(1-x) = 18.232

Now we can compute our Chi-squared statistic, which will have (2 1)(2 1) = 1 degree of freedom.

(16.232 20)2 (18.768 15)2 (15.768 12)2 (18.232 22)2


21 = + + + 3.31.
16.232 18.768 15.768 18.232
On the Chi-squared table, in the row for 1 d.o.f., this result falls below the critical value at the 0.95
level, so our p-value is more than 0.05. This is answer choice E.

CAS ST Spring 2014 Exam


10. You are given the following information about a hypothesis test:

The data is from a Normal Distribution with variance 25

H0 : = 4

Ha : = 7

H0 is rejected when X 7.25

The sample size is 80.

Calculate the probability of making a Type II error.

A. Less than 0.35


B. At least 0.35, but less than 0.45
C. At least 0.45, but less than 0.55
D. At least 0.55, but less than 0.65
E. At least 0.65

A type II error occurs when the null is accepted given that the alternative is true.
" #
X 7 7.25 7
P[X < 7.25 | = 7] = P p <p
25/80 25/80
P [N (0, 1) < 0.447213596] = (0.45) = 0.6736

which is in the range of answer choice E, but is a bit close to the cutoff. We could use interpolation to
be more sure, but even (0.44) > 0.65, so even with interpolation we would be sure to obtain a result
above 0.65.

CAS ST Spring 2014 Exam


11. You are given the following information:

X and Y are random variables with unknown means x and y

X and Y have unknown but equal variances.

Samples from X and Y are taken with sizes 40 and 50, respectively
P P
Xi Yi
40 = X = 80.1 and 50 = Y = 78.8
(Xi X)2 (Yi Y )2
P P
40 = 34.8 and 50 = 25.0

H0 : x = y

H1 : x > y

Determine the p-value of the hypothesis test.

A. Less than 0.05


B. At least 0.05, but less than 0.10
C. At least 0.10, but less than 0.15
D. At least 0.15, but less than 0.20
E. At least 0.20

For a two-sample test of means in the absence of known variances, we compute a T -statistic with
40 + 50 2 = 88 degrees of freedom,
X Y
p
2
S (1/40 + 1/50)
where
(Xi X)2 + (Yi Y )2
P P
2 34.8 40 + 25.0 50
S = = = 30.0227.
40 + 50 2 88
80.178.8 1.3
The T -statistic is = 1.162335032 = 1.118438285. Unfortunately, this means that the
30.0227(1/40+1/50)
T -table is of little help to us, since our statistic is smaller than the 0.05 level (for a one-sided test -
look in 0.1 column) critical value at all degrees of freedom from 60 on down. So we know that our
p-value is above 0.05, but not how far above it is. Our only recourse is to recall that the T -distribution
is approximated by the normal distribution for the large n case. On the normal table, a z-score of 1.118
would correspond to a probability of about 0.867, giving us an approximate p-value of 10.867 = 0.133 .
This fits into the range of answer choice C.

CAS ST Spring 2014 Exam


12. You are given the following claim observations for 2 territories:

Territory X: A random sample of 100 claims is drawn from a normal distribution with variance
10,000, and has a sample mean of 300

Territory Y : A random sample of 70 claims is drawn from a normal distribution with variance
5,000, and has a sample mean of 250

H0 : mean of Territory X = mean of Territory Y + 30


H1 : mean of Territory X > mean of Territory Y + 30

Determine the p-value of this hypothesis test.

A. Less than 5.0%


B. At least 5.0%, but less than 5.5%
C. At least 5.5%, but less than 6.0%
D. At least 6.0%, but less than 6.5%
E. At least 6.5%

Under the null-hypothesis, the mean from X minus the mean from Y is 30. The variance of the difference
in the sample means is
1 1
Var(X Y ) = Var(X) + Var(Y ) = 171.4285714.
100 70
The z-score is
X Y 30
z= = 1.527525232.
171.4285714
The p-value of this one-sided test, is 1 (1.527) 1 0.7(0.9370) 0.3(0.9357) = 0.06339 which fits
into the range of answer choice D.

CAS ST Spring 2014 Exam


13. Two six-sided dice (X and Y ) are rolled 1000 times each. The outcomes are in the table below.

Outcome X Y
1 94 157
2 244 168
3 135 197
4 158 136
5 128 220
6 241 122

An outcome of 4 or more is considered a success. Let P be the probability of a success.

H0 : PX = PY

H1 : PX 6= PY

Calculate the minimum significance level at which the null hypothesis will be rejected.

A. Less than 0.025


B. At least 0.025, but less than 0.035
C. At least 0.035, but less than 0.045
D. At least 0.045, but less than 0.055
E. At least 0.055

The proportion of successes observed for die X is 0.527, and the proportion of successes observed for die
Y is 0.478. Under the null hypothesis, the variance of each trial is the same, p(1p), where p = PX = PY .
The variance of PX PY is therefore estimated under the null as p(1 p)/1000 + p(1 p)/1000, which
0.5270.478
with an estimate of p = 0.5025 gives a z-score of = 2.19. The p-value of our
20.5025(10.5025)/1000
two-sided test is therefore 2(1 0.9857) = 0.0286 , which fits into the range of answer choice B.

CAS ST Spring 2014 Exam


14. You are given the following information about two loss severity distributions fit to a sample of 275 closed
claims:

For the Exponential distribution, the natural logarithm of the likelihood function evaluated at the
maximum likelihood estimate is -828.37.

For the Weibull distribution, the natural logarithm of the likelihood function evaluated at the
maximum likelihood estimate is -826.23.

The Exponential distribution is a subset ofthe Weibull distribution.

Calculate the significance level at which the Weibull distribution provides a better fit than the Expo-
nential distribution.

A. Less than 0.5%


B. At least 0.5%, but less than 1.0%
C. At least 1.0%, but less than 2.5%
D. At least 2.5%, but less than 5.0%
E. At least 5.0%

The natural log of the likelihood ratio times negative two is a Chi-squared statistic with 1 degree of
freedom, since the exponential random variable is a Weibull random variable with specified. Since
we are given the natural logs of the likelihood functions at their maximum values, the statistic is
2(828.37 (826.23)) = 4.28. This is between the critical values in the Chi-square table with d.o.f.
1 of 0.95 and 0.975, so our p-value is between 0.025 and 0.05, which falls into the range of answer choice
D.

CAS ST Spring 2014 Exam


15. You are given the following information:

Student Number of Practice Exams Score on Final Exam


1 8 9
2 6 8
3 5 7
4 4 6
5 3 5
6 2 10
7 1 4
8 0 3

Students are ranked in descending order for both the number of practice exams and the score on the
final exam.

Calculate Spearmans rank correlation coefficient between the number of practice exams and the score
on final exam.

A. Less than 0.0


B. At least 0.0, but less than 0.3
C. At least 0.3, but less than 0.6
D. At least 0.6, but less than 0.9
E. At least 0.9

Ranked in descending order,


Student Number of Practice Exams Score on Final Exam
1 1 2
2 2 3
3 3 4
4 4 5
5 5 6
6 6 1
7 7 7
8 8 8

Now via the MultiView calculator, entering the practice exam rank into L1 and the Exam score rank
into L2, we find that rS = 0.6428571429 , which fits into the range of answer choice D.

CAS ST Spring 2014 Exam


16. A random sample of size eight is drawn from a population in order to test the null hypothesis that the
median of the population is 2.3.

You are given the following sample values:

1.8 0.2 1.5 2.5 2.7 0.4 2.4 3.0

Calculate the Wilcoxon statistic.

A. Less than -30


B. At least -30, but less than -25
C. At least -25, but less than -20
D. At least -20, but less than -15
E. At least -15

While the provided table of values for the Wilcoxon Signed-rank test is only for the sum of either the
positive ranks or of the negative ranks, in either event a positive value, the actual Wilcoxon Signed-rank
statistic is the sum of the positive ranks minus the sum of the negative ranks, which can be a negative
number. The differences from the hypothesized median, 2.3, are

0.5 2.1 0.8 0.2 0.4 1.9 0.1 0.7

and these differences have magnitude ranks

4 8 6 2 3 7 1 5

The positive ranks sum to 11, and the negative ranks sum to 25, so that the formal Wilcoxon Signed-
rank statistic is 11 25 = 14 . This corresponds to answer choice E.

CAS ST Spring 2014 Exam


17. The probability density of Yk , the kth order statistic for a sample of size n is:
n!
gk (yk ) = [F (y)]k1 [1 F (y)]nk f (y)
(k 1)!(n k)!

Let Y1 < Y2 < Y3 < Y4 < Y5 be the order statistics of five independent observations from a unifonn
distribution on (0,10).

Calculate the variance of Y4 .

A. Less than 2
B. At least 2, but less than 4
C. At least 4, but less than 6
D. At least 6, but less than 8
E. At least 8

The density of the uniform is 1/10, and the CDF is y/10. The density of Y4 is
5!
g4 (y) = (y/10)3 (1 y/10)1 (1/10) = 2(y/10)3 (1 y/10)
3! 1!
The variable Y4 is therefore a Beta distribution with = 10, a = 4 and b = 2, From the tables,
10 4 20
E[Y4 ] = =
6 3
10245 1000
E[Y42 ] = =
67 21
 2
1000 20 200
Var(Y4 ) = = = 3.1746031746 .
21 3 63
This fits into the range of answer choice B. If you dont make the connection with the Beta distribution
things are tougher.
Z 10
E[Y4 ] = y 2(y/10)3 (1 y/10)dy Let u = y/10.
0
Z 1
= 10u 2u3 (1 u)(10du)
0
Z 1
2
= 2 10 (u4 u5 )du
0
5
!
6 1
 
2 u u 2 1 1 20
= 2 10 = 2 10 =
5 6 0 5 6 3
Z 10
E[Y42 ] = y 2 2(y/10)3 (1 y/10))dy again, u = y/10
0
Z 1
= 2 102 u2 u3 (1 u)(10du)
0
Z 1
3
= 2 10 u5 u6 du
0
1 !
u6 u7
 
3 1 1 1000
= 2 10 = 2 103 = .
6 7 0 6 7 21

CAS ST Spring 2014 Exam


18. Let Y1 < Y2 < . . . < Y12 be the order statistics of a random sample from a uniform distribution [0, 1].
Calculate the probability that 0.6 < Y12 < 0.75.

A. Less than 0.010


B. At least 0.010, but less than 0.015
C. At least 0.015, but less than 0.020
D. At least 0.020, but less than 0.025
E. At least 0.025

The 12th order stat is the maximum of the observations.

P[Y12 t] = t12
P[0.6 Y12 0.75] = P[Y12 0.75] P[Y12 0.6]
= 0.7512 0.612 = 0.02949957 ,

which corresponds to answer choice E.

CAS ST Spring 2014 Exam


19. You are given the following sales information from three car dealers:

Dealer Number of Cars Sold Average Sales Price (000s) SS(E)


A 6 23.8 6.83
B 4 25.3 2.75
C 5 27.4 37.20

You wish to test whether the average sales prices are the same among the dealers:

H0 : A = B = C (The mean sales price between the three dealers is the same)
H1 : The mean sales price between the three dealers is not the same.

Create an ANOVA table and assume the total sum squares, SS(TO) = 81.6

Which of the following statements is TRUE?

A. Reject H0 when = 0.05 and reject H0 when = 0.01


B. Reject H0 when = 0.05 but do not reject H0 when = 0.01
C. Do not reject H0 when = 0.05 but reject H0 when = 0.01
D. Do not reject H0 when = 0.05 and do not reject H0 when = 0.01
E. None of the above

We are given the sums of squares errors for each of the three dealerships, so to find the total sum of
squares errors, we add these together to get 46.78. The sum of squares treatement is the total less the
error, 81.6 46.78 = 34.82.

The F -statistic is
SSTr
31
F2,12 = SSE
= 4.466
153
Looking at the second column of the F -table, we see that this result is between the 0.05 critical value
(3.88) and the 0.01 critical value (6.93), so we will reject at the 0.05 level, but not at the 0.01 level.
Answer choice B.

CAS ST Spring 2014 Exam


20. The model yi = 0 + 1 xi + i was fit using 6 observations. The estimated parameters are as follows:

0 = 2.31

1 = 1.15

0 = 0.057

1 = 0.043

The following hypothesis test is performed:

H0 : 1 = 1

H1 : 1 6= 1

Determine the minimum significance level at which the null hypothesis would be rejected.

A. Less than 0.01


B. At least 0.01, but less than 0.02
C. At least 0.02, but less than 0.05
D. At least 0.05, but less than 0.10
E. At least 0.10

We are accustomed to using = 0 , = 1 , and s = 1 so translate the formulas carefully.

1 1 1.15 1
T62 = = = 3.48837209
1 0.043

In the T -table, the result lies between the 0.05 critical value (2.776) and the 0.02 critical value (3.747)
of the fourth row. Our T -table naturally tests two sided tests, which we want here, so we would reject
at the 0.05 level, but not at the 0.02 level, corresponding to answer choice C.

CAS ST Spring 2014 Exam


21. For i = 1, 2, 3, 4 let Xi denote the speed of cars passing location i.

Assume that the distribution of each Xi is normal with mean i and common variance 2

H0 : 1 = 2 = 3 = 4 (The means are all equal)

H1 : the means are not all equal

xij is observation j at location i

You have the following observations:


P5 P5 2
Location Speed by Car Observed j=1 xij j=1 xij
1 52.4 55.7 60.2 69.9 59.4 297.6 17,886.66
2 62.5 65.1 56.5 52.8 69.0 305.9 18,885.35
3 61.0 63.5 84.0 68.1 77.6 354.2 25,468.62
4 63.0 52.1 65.2 59.5 52.8 292.6 17,262.54
Totals 1250.3 79,503.17

Determine the F -statistic used to test H0 .

A. Less than 2.5


B. At least 2.5, but less than 2.7
C. At least 2.7, but less than 2.9
D. At least 2.9, but less than 3.1
E. At least 3.1

This is where memorizing the short ANOVA formulas is super helpful.


2
X 1 X
SST = x2ij xij
n
i,j i,j
1
1250.32 = 1340.665

= 79503.17
20
2 2
X1 X 1 X
SSTr = xij xij
5 20
i j i,j
1 1
297.62 + 305.92 + 354.22 + 292.62 1250.32 = 480.0895
 
=
5 20
SSE = 1340.665 480.0895 = 860.576.
480.0895
41
F3,16 = 860.576 = 2.975306461
204

This falls into the range of answer choice D.

CAS ST Spring 2014 Exam


22. You are given the following information for an insurance policy:

Monthly claim frequencies follow a Poisson process with parameter .

The prior distribution of follows a gamma distribution with parameters = 3 and = 2.

In the first month, a policy had 27 claims.

Calculate the posterior mean monthly claims for this policy.

A. Less than 9
B. At least 9, but less than 13
C. At least 13, but less than 17
D. At least 17, but less than 21
E. At least 21

This is a Poisson/Gamma pair with k = 27 claims observed in m = 1 period. The posterior is Gamma
with 0 = 3 + 27 = 30, and = 1/2+1
1
= 2/3. The mean of this posterior is 30 23 = 20 , which leads to
answer choice D.

CAS ST Spring 2014 Exam


23. You are given the following information:

X1 , . . . , X5 are a random sample from a Poisson distribution with parameter , where follows a
gamma distribution with parameters = 2 and .

The mean of this Poisson-gamma conjugate pair can be represented as a weighted average of the
maximum likelihood estimator for the mean and the mean of the prior distribution.

Let WMLE be the weight assigned to the maximum likelihood estimator.

The maximum likelihood estimate for the mean is 1.2.

The variance of the prior gamma is 8.

Calculate WMLE .

A. Less than 0.60


B. At least 0.60, but less than 0.70
C. At least 0.70, but less than 0.80
D. At least 0.80, but less than 0.90
E. At least 0.90

From bullet one, the prior distribution is Gamma with = 2. From bullet five, the variance of this gamma
is 8, which allows us to compute .

Var(Prior) = 2 ()2
8 = 3 22 42 = 22
= 2.

The average of the prior is therefore 2 2 = 4.

Im assuming that the mean of the conjugate pair refers to the mean of the posterior or predicitive
distribution, which are the same for the Poisson/Gamma case. Since the MLE for the mean is X, we
find that the number of exposures is n = 5 and the number of claims observed is nX = 5 1.2 = 6.
The posterior is therefore Gamma with 0 = 2 + 6 and 0 = 1/2+5
1
= 2/11. The mean of the posterior is
8 2/11 = 16/11.

To find the weight WMLE , set up the weighted average, set it equal to 16/11, and solve.

16/11 = WMLE (1.2) + (1 WMLE )4


(4 1.2)WMLE = 4 16/11
4 16/11
WMLE = = 10/11 = 0.90
4 1.2
This result lies in the range of answer choice E.

CAS ST Spring 2014 Exam


24. You are given the following information about a random variable X:

X follows the Normal distribution with parameters N (, 2 ) where 2 , the variance, is known to
be 2.

is assumed to follow a Normal distribution with parameters N (1, 4) prior to gathering a random
sample

A random sample of size 10 is collected to develop a posterior estimate of and the posterior
estimate of is equal to 1.2

Calculate the sample mean of the random sample.

A. Less than 1.25


B. At least 1.25, but less than 1.30
C. At least 1.30, but less than 1.35
D. At least 1.35, but less than 1.40
E. At least 1.40

This is a Normal/Normal pair. For a N (, ) with N (, a), the posterior mean is found by computing
n
z n+/a , and then using the formula
(1 z) + z(X)
n 10
Here, z n+/a = 10+2/4 = 20/21, and given that the posterior mean turns out to be 1.2,

1.2 = (1 z) + z(X)
1.2 = 1(1 20/21) + 20/21(X)
25.2 = 1 + 20X
X = 1.21

which fits into the range of answer choice A.

CAS ST Spring 2014 Exam


25. You are given the following information:

Y follows a binomial distribution with parameters n and p = .

The prior distribution of follows a beta distribution with parameters = 4 and = 6


P
A sample of size 15 is drawn from Y with yi = 10.

Calculate the estimate of for the posterior distribution.

A. Less than 0.50


B. At least 0.50, but less than 0.60
C. At least 0.60, but less than 0.70
D. At least 0.70, but less than 0.80
E. At least 0.80

This is a Beta/Binomial pair. The number of exposures is 15 n, with k = 10 claims observed, to put
things in our usual language. The posterior is Beta with a0 = 4 + 10, b0 = 6 + 15 n 10. The Bayesian
14 14
estimate for is the mean of the posterior, 14+15n4 = 10+15n . At this point you have some thinking
to do. If the CAS meant for this to be a Bernoulli distribution instead of a Binomial, then n = 1, the
result is 0.56, and this fits into answer choice B. This seems to be the most likely interpretation for this
problem.

CAS ST Spring 2014 Exam

Você também pode gostar